Difference between revisions of "2018 AMC 8 Problems/Problem 7"

(Created page with "==Problem 7== The <math>5</math>-digit number <math>\underline{2}</math> <math>\underline{0}</math> <math>\underline{1}</math> <math>\underline{8}</math> <math>\underline{U}</...")
 
(Problem 7)
Line 3: Line 3:
  
 
<math>\textbf{(A) }1\qquad\textbf{(B) }3\qquad\textbf{(C) }5\qquad\textbf{(D) }6\qquad\textbf{(E) }7</math>
 
<math>\textbf{(A) }1\qquad\textbf{(B) }3\qquad\textbf{(C) }5\qquad\textbf{(D) }6\qquad\textbf{(E) }7</math>
 +
{{AMC8 box|year=2018|num-b=6|num-a=8}}

Revision as of 11:57, 21 November 2018

Problem 7

The $5$-digit number $\underline{2}$ $\underline{0}$ $\underline{1}$ $\underline{8}$ $\underline{U}$ is divisible by $9$. What is the remainder when this number is divided by $8$?

$\textbf{(A) }1\qquad\textbf{(B) }3\qquad\textbf{(C) }5\qquad\textbf{(D) }6\qquad\textbf{(E) }7$

2018 AMC 8 (ProblemsAnswer KeyResources)
Preceded by
Problem 6
Followed by
Problem 8
1 2 3 4 5 6 7 8 9 10 11 12 13 14 15 16 17 18 19 20 21 22 23 24 25
All AJHSME/AMC 8 Problems and Solutions